Calculating the sum of a sequence

  • Thread starter Thread starter goraemon
  • Start date Start date
  • Tags Tags
    Sequence Sum
Click For Summary
The discussion centers on calculating the sum of the series ∑(4/(-3)^n - 3/3^n) as n approaches infinity. The user initially struggles with identifying the series type but recognizes it can be separated into two geometric series. By applying the geometric sum formula, they compute the sums for both series starting from n=0. The final calculation yields a result of -3/2, which is confirmed as correct by another participant. The conversation highlights the application of geometric series principles in solving the problem.
goraemon
Messages
65
Reaction score
4

Homework Statement


Compute \sum\frac{4}{(-3)^n}-\frac{3}{3^n}
as n begins from 0 and approaches infinity


Homework Equations




The Attempt at a Solution



I'm just getting started on sequences and series, and so far learned about the limit test, comparison test, arithmetic / geometric series and the like. Oh and a little bit about telescoping series. I'm just not sure how to approach this question. It doesn't look like a geometric or arithmetic series, and I tried to compute the first few terms to see whether it qualifies as a telescoping series, but it doesn't look like it. Thanks for any help.
 
Physics news on Phys.org
Note that ##\sum\frac{4}{(-3)^n}-\frac{3}{3^n}=\sum\frac{4}{(-3)^n}-\sum\frac{3}{3^n}=4\sum\frac{1}{(-3)^n}-3\sum\frac{1}{3^n}.## Apply geometric sum formula.
 
xiavatar said:
Note that ##\sum\frac{4}{(-3)^n}-\frac{3}{3^n}=\sum\frac{4}{(-3)^n}-\sum\frac{3}{3^n}=4\sum\frac{1}{(-3)^n}-3\sum\frac{1}{3^n}.## Apply geometric sum formula.

Ah ok...so then since n begins at zero...the answer would be...
##4\sum\frac{1}{(-3)^n}-3\sum\frac{1}{3^n}=4*\frac{1}{1-\frac{-1}{3}}-3*\frac{1}{1-\frac{1}{3}}=4*\frac{3}{4}-3*\frac{3}{2}=\frac{-3}{2}##

Is that right? Thanks!
 
Yes. That is correct.
 
  • Like
Likes 1 person
Question: A clock's minute hand has length 4 and its hour hand has length 3. What is the distance between the tips at the moment when it is increasing most rapidly?(Putnam Exam Question) Answer: Making assumption that both the hands moves at constant angular velocities, the answer is ## \sqrt{7} .## But don't you think this assumption is somewhat doubtful and wrong?

Similar threads

  • · Replies 1 ·
Replies
1
Views
2K
Replies
2
Views
5K
Replies
4
Views
2K
  • · Replies 4 ·
Replies
4
Views
2K
  • · Replies 7 ·
Replies
7
Views
2K
  • · Replies 6 ·
Replies
6
Views
2K
  • · Replies 3 ·
Replies
3
Views
1K
  • · Replies 7 ·
Replies
7
Views
1K
  • · Replies 2 ·
Replies
2
Views
1K
  • · Replies 4 ·
Replies
4
Views
3K